ROSK PANCE QBANK PSYCH

Pataasin ang iyong marka sa homework at exams ngayon gamit ang Quizwiz!

You finish taking a history of a patient at risk for suicide. Which of the following would suggest the highest likelihood of committing suicide? (A Age over 85 years B Cohabitation with parents C Married with children D Tactile hallucinations)

Age over 85 years (Suicide is the tenth most common cause of death in the US, and is in the top 3 most common causes of death in adolescents and young adults. It is more common in non-Hispanic White and American Indian men between the ages of 45-59 years. The greatest suicide rate is in elderly white males over 85 years old (A). Frequently, the primary care provider needs to assess the risk of suicide. Activities associated with committing suicide include buying a rope or firearm, filling out a will, contacting old friends and writing a suicide note. Common characteristics of those who commit suicide include a sense of violation, a preoccupation with death, a lack of humor, distractibility, few friends or family members, hopelessness and an inability to help themselves or a sense that there is nothing to help their condition. When evaluating for suicide risk, identify any of the above characteristics. Also, determine if suicidal ideation (thoughts of hurting oneself) is present, and if so, determine if a suicide plan is in place. The more detailed the plan, the higher likelihood of suicide. Also, determining the purpose of suicide can help to gauge the seriousness of the action. Any question of suicide must be coupled to an inquiry into the possibility of homicide, as both situations represent aggression. If there is a tendency toward aggression towards others (homicide), the likelihood of aggression toward oneself (suicide) is high. Furthermore, the presence of any of the following supports an actual committal of suicide: definite plan, activities one does before dying (saying goodbye to friends), family history of suicide, possession of a firearm, current substance use, depression of any type, anxiety of any type, command hallucinations, recent discharge from a psychiatric hospital, severe immediate loss such as an unexpected divorce, or isolation (patient is alone).)

Question: What are some common side effects of modafinil?

Answer: Anxiety, headache, nausea, and nervousness. (Narcolepsy Sx: daytime sleepiness, falling asleep at inappropriate times with minimal warning, sleep attacks, cataplexy (emotionally triggered muscle weakness), hypnagogic or hypnopompic hallucinations (vivid hallucinations as patients are falling asleep or waking up), and sleep paralysis upon awakening Dx: polysomnography, multiple sleep latency test Management begins with sleep specialist referral, may include scheduled naps, stimulants, modafinil May coexist with other sleep disorders)

Question: Which selective serotonin reuptake inhibitor has the greatest affinity for causing QTc prolongation?

Answer: Citalopram.

Question: Opioid use via contaminated needles warrants testing for which viral infections?

Answer: HIV and hepatitis.

Question: What is the prevailing theory of the mechanism of action of bright light therapy for depression?

Answer: Modulation of neurotransmitters such as serotonin.

Question: Which gender is more likely to experience all 5 stages of grief?

Answer: Women are more likely than men to experience all 5 stages.

Which of the following is true regarding anorexia nervosa? (A Affects men more than women B Antidepressants are an effective treatment C Associated with a body image disturbance D Menorrhagia is an early symptom)

Associated with a body image disturbance (Eating disorders are the third leading chronic illness, after obesity and asthma, in adolescent girls. An individual with anorexia nervosa refuses to maintain a minimally normal body weight, is fearful of gaining weight, and exhibits a distorted body self-image. The patient's body image is the predominant measure of self-worth, along with denial of the seriousness of the illness. The long-term mortality rate for anorexia nervosa is 6% to 20%, the highest rate for any psychiatric disorder. Anorexia is associated with amenorrhea, depression, fatigue, weakness, hair loss, bone pain, constipation, and abdominal pain. Signs include brittle hair and nails, dry, scaly skin, loss of subcutaneous fat, fine facial and body hair (lanugo), and breast and vaginal atrophy. A prime objective in assessment is to distinguish "normal dieters" from individuals with eating disorders. Another important aspect to the evaluation is to exclude certain medical conditions such as inflammatory bowel disease, hyperthyroidism, chronic infection, diabetes mellitus, and Addison disease. Indications for inpatient management include extremely low body weight (< 75% of expected) or rapid weight loss; severe electrolyte imbalances, cardiac disturbances, or other acute medical disorders; severe or intractable purging; psychosis or a high risk of suicide; and symptoms refractory to outpatient treatment. A multidisciplinary team that includes a primary physician, mental health professional, and registered dietician should manage patients with eating disorders. Refeeding should occur in a monitored setting due to the risk of dysrhythmia and refeeding syndrome.)

A 52-year-old business executive presents to the ED with diaphoresis, tachycardia, visual hallucinations, and recent seizure. The patient states that he drinks alcohol daily but he is trying to quit. His last drink was one day ago. Which of the following is the most appropriate medication at this time? (A Chlordiazepoxide B Haloperidol C Phenytoin D Quetiapine)

Chlordiazepoxide (Abrupt cessation of alcohol intake in a chronic alcoholic can lead to alcohol withdrawal syndrome. A history and physical is usually all that is necessary to make a diagnosis. Ethanol withdrawal is characterized by autonomic hyperactivity (diaphoresis, tachycardia), hand tremor, insomnia, nausea, vomiting, hallucinations (visual), anxiety psychomotor agitation and seizures. Treatment includes monitoring, serial Clinical Institute Withdrawal Assessment for Alcohol (revised) (CIWA-Ar) testing and long acting benzodiazepines like chlordiazepoxide or diazepam. Haloperidol (B) can be used to treat agitation and hallucinations of alcohol withdrawal syndrome, however, it can lower the seizure threshold. This would not be prudent in a patient with possible recent seizure activity. Phenytoin (C) does not treat alcohol withdrawal syndrome seizures, however, it may be considered as adjuvant therapy in a patient with a documented seizure disorder. The atypical antipsychotic quetiapine (D) is not used in the treatment of alcohol withdrawal hallucinations.)

A 32-year-old man presents to the psychiatry clinic due to an excessive fear of needles. This fear has led him to avoid necessary medical care because he has panic attacks at the thought of having blood drawn. He recently had to be sedated in the emergency department to have a peripheral IV started. Vital signs include HR of 80 bpm, BP of 120/80 mm Hg, RR of 20/minute, oxygen saturation of 98% on room air, and T of 98.6°F. Physical examination reveals a regular rate and rhythm and lungs clear to auscultation. Which of the following is the recommended treatment for the suspected condition? AExposure therapy BLorazepam CQuetiapine DSertraline ETrauma-focused psychotherapy

Correct Answer ( A ) Explanation: A phobia is defined as an excessive and intense fear of an object or situation that frequently leads to avoidant behaviors and considerable distress. The feared object or situation is called the stimulus. The pathogenesis of phobias has environmental and genetic components. Individuals often develop phobias after negative or traumatic experiences with the stimulus. Common specific phobias encountered in clinical practice are phobias of flying, elevators, enclosed spaces, heights, receiving an injection, seeing blood, and specific animals (e.g., spiders, dogs, snakes). Individuals are not diagnosed with a specific phobia if they have a significant fear but do not encounter the phobic situation and are not distressed by having the fear. Specific phobias are diagnosed according to the criteria in the fifth edition of the Diagnostic and Statistical Manual of Mental Disorders. The recommended initial treatment of specific phobias is cognitive behavioral therapy that includes exposure therapy. The most effective type of exposure is real-life exposure rather than imaginal exposure. However, imaginal exposure can be useful if the phobic stimulus is too costly or infrequent to recreate. The exposure treatment can be provided in one 2-3 hour session or several 90-minute sessions. It is important to provide cognitive strategies to help patients tolerate the anxiety induced by exposures. These strategies include providing education to correct misunderstood aspects of the stimulus and helping patients identify maladaptive thoughts and appraisals that trigger phobic fear.

A 52-year-old business executive presents to the ED with diaphoresis, tachycardia, visual hallucinations, and recent seizure. The patient states that he drinks alcohol daily but he is trying to quit. His last drink was one day ago. Which of the following is the most appropriate medication at this time? AChlordiazepoxide BHaloperidol CPhenytoin DQuetiapine

Correct Answer ( A ) Explanation: Abrupt cessation of alcohol intake in a chronic alcoholic can lead to alcohol withdrawal syndrome. A history and physical is usually all that is necessary to make a diagnosis. Ethanol withdrawal is characterized by autonomic hyperactivity (diaphoresis, tachycardia), hand tremor, insomnia, nausea, vomiting, hallucinations (visual), anxiety psychomotor agitation and seizures. Treatment includes monitoring, serial Clinical Institute Withdrawal Assessment for Alcohol (revised) (CIWA-Ar) testing and long acting benzodiazepines like chlordiazepoxide or diazepam. Haloperidol (B) can be used to treat agitation and hallucinations of alcohol withdrawal syndrome, however, it can lower the seizure threshold. This would not be prudent in a patient with possible recent seizure activity. Phenytoin (C) does not treat alcohol withdrawal syndrome seizures, however, it may be considered as adjuvant therapy in a patient with a documented seizure disorder. The atypical antipsychotic quetiapine (D) is not used in the treatment of alcohol withdrawal hallucinations.

A 49-year-old man presents to the Emergency Department complaining of sweating and tremors. The patient drinks a bottle of liquor per day and stopped suddenly because of a pending court case. His last alcoholic drink was 3 days ago. On physical examination, his blood pressure is 168/105 mm Hg, pulse rate is 106/minute, respirations are 22/minute, and temperature is 99.3°F. The patient appears agitated and restless with a visible tremor of bilateral hands. The triage team ordered folic acid, thiamine, and a multivitamin. Which of the following is the most appropriate disposition? AAdmit the patient and start diazepam BAdmit the patient and start disulfiram CDischarge the patient with a prescription for diazepam DDischarge the patient with a prescription for disulfiram

Correct Answer ( A ) Explanation: Admit the patient and start diazepam is the correct disposition because this patient is suffering from alcohol withdrawal, which potentially can be fatal. Withdrawal symptoms occur when a patient has alcohol use disorder and has developed a tolerance to alcohol, where an increased amount of alcohol is needed to achieve the desired effect. When tolerance has developed, cessation leads to withdrawal. Early symptoms of alcohol withdrawal include anxiety, irritability, headache, tremor, tachycardia, hypertension, hyperthermia, and hyperactive reflexes. Seizures (usually grand mal) can develop between 12-24 hours after withdrawal starts. After 24-72 hours, life-threatening delirium tremens may occur, which manifests with signs of altered mental status, hallucinations and marked autonomic instability. Treatment of alcohol withdrawal involves giving a benzodiazepine (e.g. diazepam) until symptoms lessen and then tapering the dosage over days to weeks. Thiamine, folic acid, and vitamin B12 are also administered and any electrolyte abnormalities are corrected (typically low potassium and magnesium). Following withdrawal, the patient should be referred to support groups. Long term medication used to deter use of alcohol include naltrexone, disulfiram, and acamprosate.

Which of the following is the most common type of child abuse? ANeglect BPhysical CPsychological DSexual

Correct Answer ( A ) Explanation: Child abuse can be defined as physical, sexual or emotional maltreatment, either via commission or omission, that results in actual, potential or threatened harm to a child. The most common form of child abuse is neglect. Neglect is defined as the failure of a parent or responsible adult to provide food, clothing, shelter medical care and supervision. A history of childhood abuse can lead to several chronic conditions, especially psychiatric issues such as anxiety, depression, acting-out, post-traumatic stress disorder, chronic pain, neurocognitive disorders, personality disorders and disorganized attachment. Up to 80% of abused children have one psychiatric disorder by age 21 years.

A previously healthy 52-year-old woman presents to your office with a complaint of diffuse pruritus. She is concerned about insects under her skin. She brought a sample of her skin mixed with debris that she believes are the insects. She is requesting that you send it to the lab. Physical exam reveals scattered ulcers and erosions in varying stages of healing with excoriations all over her body. Which of the following is the most effective management? AEstablish a strong therapeutic alliance BPermethrin cream as needed CReferral to psychiatry DReferral to substance abuse treatment

Correct Answer ( A ) Explanation: Delusions of parasitosis (DoP) is a delusional disorder involving the firm belief by the patient that the pruritus is caused by an infestation of insects or parasites. Patients present with self-inflicted skin manifestations from scratching or digging and may bring a sample of debris, lint, or pieces of skin that they say contain the insects or parasites. Diagnosis involves excluding any true skin infestations, such as scabies, as well as ruling out systemic disease that may cause pruritus. Once a physical etiology has been excluded, diagnosis is through meeting criteria established by the Diagnostic and Statistical Manual of Mental Disorders (DSM). Initial management is by establishing a strong therapeutic alliance with the patient and respecting the patient's autonomy in all encounters. First-line pharmacologic treatment is with antipsychotic medications. In treating DoP, it is important that the medical provider does not offer treatment that will strengthen the patient's delusion, such as prescribing permethrin as needed (B). A referral to psychiatry (C) is an appropriate part of the management of DoP, but should only be done once a therapeutic alliance has been established since initially patients will not accept that their symptoms are due to a psychiatric etiology. Substance abuse should be ruled out as a possible cause of symptoms, but this may be done as part of the history in the outpatient setting and a referral to substance abuse treatment (D) is only necessary if the patient has a substance abuse disorder per DSM-5.

A 29-year-old woman presents to your office with concerns about worsening anxiety. Which additional finding would suggest a diagnosis of generalized anxiety disorder rather than panic disorder? AAnxiety occurring more days than not for at least 6 months BChanges in personality CExcessive concern about medically unexplained symptoms DFrequent periods of intense fear

Correct Answer ( A ) Explanation: Generalized anxiety disorder (GAD) is a common psychiatric disorder often seen in the primary care setting. It is characterized by excessive and persistent worrying that occurs more days than not for ≥ 6 months. Other clinical manifestations include insomnia, headaches, difficulty relaxing, and fatigue. The anxiety symptoms experienced with GAD are difficult to control and cause significant distress and impairment in activities of daily living. GAD is two times more common in women than in men and is the most common psychiatric disorder seen in older adults. Diagnosis is determined using the fifth edition of the Diagnostic and Statistical Manual of Mental Disorders (DSM-5) criteria. Recommended treatment is with a combination of pharmacotherapy and psychotherapy. Changes in personality (B) and frequent periods of intense fear (D) occur in patients with panic disorder. Excessive concern about medically unexplained symptoms (C) is seen with both GAD and illness anxiety disorder (hypochondriasis). Individuals with GAD generally worry about multiple concerns, whereas individuals with illness anxiety disorder (hypochondriasis) are focused on their perceived medical issues.

You finish taking a history of a patient at risk for suicide. Which of the following would suggest the highest likelihood of committing suicide? AAge over 85 years BCohabitation with parents CMarried with children DTactile hallucinations

Correct Answer ( A ) Explanation: Suicide is the tenth most common cause of death in the US, and is in the top 3 most common causes of death in adolescents and young adults. It is more common in non-Hispanic White and American Indian men between the ages of 45-59 years. The greatest suicide rate is in elderly white males over 85 years old (A). Frequently, the primary care provider needs to assess the risk of suicide. Activities associated with committing suicide include buying a rope or firearm, filling out a will, contacting old friends and writing a suicide note. Common characteristics of those who commit suicide include a sense of violation, a preoccupation with death, a lack of humor, distractibility, few friends or family members, hopelessness and an inability to help themselves or a sense that there is nothing to help their condition. When evaluating for suicide risk, identify any of the above characteristics. Also, determine if suicidal ideation (thoughts of hurting oneself) is present, and if so, determine if a suicide plan is in place. The more detailed the plan, the higher likelihood of suicide. Also, determining the purpose of suicide can help to gauge the seriousness of the action. Any question of suicide must be coupled to an inquiry into the possibility of homicide, as both situations represent aggression. If there is a tendency toward aggression towards others (homicide), the likelihood of aggression toward oneself (suicide) is high. Furthermore, the presence of any of the following supports an actual committal of suicide: definite plan, activities one does before dying (saying goodbye to friends), family history of suicide, possession of a firearm, current substance use, depression of any type, anxiety of any type, command hallucinations, recent discharge from a psychiatric hospital, severe immediate loss such as an unexpected divorce, or isolation (patient is alone).

Which one of the following cardiac rhythm abnormalities is most common in patients with anorexia nervosa? AAtrial fibrillation BSinus bradycardia CSinus tachycardia DVentricular fibrillation

Correct Answer ( B ) Explanation: Anorexia nervosa is characterized by restriction of food intake resulting in low body weight, intense fear of gaining weight or becoming fat, and disturbance of body image. It occurs most often in adolescent females and is often accompanied by depression and other comorbid psychiatric disorders. For low-weight patients with anorexia nervosa, virtually all physiologic systems are affected, ranging from hypotension and osteopenia to life-threatening dysrhythmias, often requiring emergent assessment and hospitalization for metabolic stabilization. Sinus bradycardia is almost universally present in patients with anorexia nervosa. It is hypothesized that this is due to vagal hyperactivity resulting from an attempt to decrease the amount of cardiac work by reducing cardiac output. As cachexia progresses, patients with anorexia nervosa lose strength and endurance, move more slowly, and demonstrate decreased performance in sports. Overuse injuries and stress fractures can occur. Bradycardia, orthostatic hypotension, and palpitations may progress to potentially fatal dysrhythmias. The focus of initial treatment for patients who have anorexia nervosa with cachexia is restoring nutritional health, with weight gain as a surrogate marker. Feeding tubes may be needed in severe cases when the patient has a high resistance to eating. Refeeding syndrome can occur in a malnourished individual when a rapid increase in food intake results in dramatic fluid and electrolyte shifts, and is potentially fatal.

A four-year-old boy is seen in your office for routine physical. His mother is concerned that he is not speaking like a normal four year old. On further evaluation, you notice he has poor eye contact and rapid hand movements, and he is preoccupied by repeatedly spinning the wheels on the toy cars he brought with him. What is the most likely diagnosis? AAttention deficit hyperactive disorder BAutism spectrum disorder CExpressive speech delay DObsessive compulsive disorder

Correct Answer ( B ) Explanation: Autistic disorder is a clinical diagnosis. The core features include impaired social interactions, communication, and behavior. Early social skills deficits include poor eye contact, lack of interactive play, and lack of interest in other children. Children can range from being nonverbal to having impaired speech. Their speech may have odd intonation and be characterized by echolalia. Language delay becomes a concern when there is lack of babbling or gestures by 12 months, lack of single words by 16 months, absent two-word phrases by 24 months, and any loss of language skills at any time. Children with autism often have abnormal play skills. They have a preoccupation with parts of objects like the child in this vignette. These children are withdrawn and can spend hours playing alone. Intellectual skills can vary, and some children may show normal or even accelerated development in certain areas.

A 19-year-old man presents with his parents who state that their son has shown signs of impulsivity, pressured speech, racing thoughts, and a decreased need for sleep over the last three to four weeks that has caused him to miss school. What is the most likely diagnosis? AAttention-deficit/hyperactivity disorder BBipolar disorder CMajor depressive disorder DSchizophrenia

Correct Answer ( B ) Explanation: Bipolar disorder is a chronic mood disorder characterized by the presence of mania (bipolar I disorder) or hypomania and depression (bipolar II disorder). Manic episodes are distinct periods of abnormally and persistent moods that can be euphoric, expansive, or irritable. Manic patients often have greatly inflated self-esteem, confidence, decreased need for sleep, pressured speech, racing or crowded thoughts, distractibility, increased involvement in goal-directed activities (e.g., starting many projects but being unable to finish any), hypersexuality, and excessive involvement in pleasurable activities with a high potential for painful consequences. Despite mania being the defining characteristic of the disease, depressed moods tend to predominate, with bipolar I patients experiencing a 3:1 ratio of depression to mania over the course of the illness. Hypomania consists of manic episodes that do not lead to social or occupational dysfunction. Pharmacologic treatment for bipolar consists of mood stabilizers such as lithium, valproic acid, and carbamazepine. Bipolar disorder is associated with a high rate of suicide (up to 15%) and several comorbidities such as substance abuse and anxiety disorders.

Which of the following psychotherapy techniques is most effective for panic disorder? AAcceptance and commitment therapy BCognitive behavioral therapy CDialectical behavior therapy DInterpersonal therapy

Correct Answer ( B ) Explanation: Cognitive behavioral therapy is a psychotherapeutic technique that focuses on a person's thoughts and beliefs and how it influences their actions, and ways to change their thinking to become more positive and healthy. Panic disorder is characterized by an intense fear along with physiological symptoms of chest pain, shortness of breath, nausea, vomiting, and headache. This is accompanied by fear of future attacks and behavioral changes in order to prevent the onset of a panic attack. Theorists believe that panic disorder is an acquired fear of those aroused bodily sensations. Cognitive behavioral therapy allows a patient to expose those fears, analyze why they are present, and teaches them ways to cope in anxiety-provoking situations. By educating a patient, teaching self-monitoring of emotions, exposing a patient to their fear, and re-learning behavioral techniques, cognitive behavioral therapy is effectively able to decrease the symptoms of panic disorder. Acceptance and commitment therapy (A) is a form of mindfulness therapy that focus on acceptance of a problem and ways to develop strategies to cope with them. While it can be used as a psychotherapy for panic disorder, it has not yet been proven to be as effective as cognitive behavioral therapy. Dialectical behavior therapy (C) is when two opposing views are discussed until a balance of the two extremes is reached. This allows the patient to feel validated in their belief while also concluding the need to change. This form of therapy is typically used in patients suffering from borderline personality disorder or suicidal ideation. Interpersonal therapy (D) identifies how a person interacts with others and works to improve communication. This form of psychotherapy is often used to treat major depression disorder and has not been shown to be more effective than cognitive behavioral therapy in those suffering from panic disorder.

Which of the following is most consistent with alcohol withdrawal? AAgitation and increased appetite BDelirium, hallucinations, and tremor CDepressed mood with diarrhea DDilated pupils, runny nose, and watery eyes

Correct Answer ( B ) Explanation: Major alcohol withdrawal, also known as delirium tremens, occurs in less than 5% of alcoholics in withdrawal. Delirium tremens is usually preceded by minor withdrawal symptoms, but may occur at any time in the course of withdrawal. The delirium often begins 3 to 4 days after the last drink and is characterized by a marked change in sensorium with agitation, visual hallucinations, and severe disorientation. Most alcoholics who withdraw from alcohol experience minimal symptoms, such as sleep disturbance or anxiety. A small number may have tremulousness, agitation, diaphoresis, and cognitive impairment. The tremors or shakes typically begin 12 to 14 hours after a period of heavy drinking and are usually noted in the early morning. Other symptoms of withdrawal include nausea, vomiting, poor oral intake, sweats, and anxiety. Seizures during alcohol withdrawal tend to occur as one isolated seizure or a brief cluster of seizures. Treatment is supportive (eg. intravenous hydration, nutritional supplementation, close monitoring) as well as benzodiazepines for control of psychomotor agitation and seizures.

A 55-year-old woman with a history of schizoaffective disorder presents to your office to discuss her current treatment regimen. She was started on olanzapine at her last visit and wants to know what type of monitoring is recommended while taking this medication. Which of the following is the most appropriate diagnostic test to order? AAbsolute neutrophil count BLipid profile CRenal panel DThyroid stimulating hormone

Correct Answer ( B ) Explanation: Olanzapine is a second-generation antipsychotic used in the treatment of schizoaffective disorder, schizophrenia, bipolar disorder, and treatment-resistant depression. Second-generation antipsychotics, also called atypical antipsychotics, are used as first-line therapy due to the decreased risk of extrapyramidal side effects and tardive dyskinesia that can occur with the use of first-generation antipsychotics. Schizoaffective disorder is a mental illness characterized by features of both schizophrenia and a mood disorder. Medication selection generally includes an antipsychotic plus an agent to address either the depression or manic subtypes of the disorder. Patients taking a second-generation antipsychotic need to be carefully monitored for the development of metabolic syndrome, as weight gain, dyslipidemia and diabetes mellitus are side effects of these agents. Both short- and long-term monitoring of weight, blood pressure, lipid profile, fasting glucose and waist circumference is recommended for patients taking any antipsychotic medications. Pre-existing conditions such as obesity or prediabetes should be factored into the decision about which second-generation antipsychotic agent to use, as some have a higher risk for the development of metabolic syndrome than others. Olanzapine and clozapine have the highest risk, while aripiprazole, ziprasidone, lurasidone and pimavanserin carry the lowest risk.

A 7-year-old boy presents to the office with his mother for evaluation of his angry and defiant behavior that has been steadily worsening over the past year. The mother is teary and exasperated, explaining that it is a battle every day to get him to do normal tasks like getting ready for school or eating meals since he is often argumentative and does not do what he is told. He constantly blames her and her husband for making his life miserable. You suspect oppositional defiant disorder. Assessing for the presence of what common co-occurring disorder is key in effective treatment? AAntisocial personality disorder BAttention-deficit/hyperactivity disorder CAutism spectrum disorder DDisruptive mood dysregulation disorder

Correct Answer ( B ) Explanation: Oppositional defiant disorder is described in patients who portray behaviors that show disobedience, defiance, and hostility toward authority figures. About half of children with attention-deficit/hyperactivity disorder have oppositional defiant disorder and must be evaluated and treated with a combination of therapy and stimulants. Children are more likely to develop oppositional defiant disorder if they grow up in a home where the child's behavior is treated with harsh, punitive, and inconsistent parenting. Parent guidance and therapy for the child is essential for treatment, with early intervention being more effective than waiting for the child to outgrow these behaviors. Parent management therapy involves training parents to change their own behaviors which will help alter their children's problem behavior at home. Group therapy for adolescent is also effective. Children with untreated oppositional defiant disorder are prone to develop conduct disorder, which exhibits more severe symptoms such as stealing, physical violence, sexually inappropriate behaviors, destruction of property, and aggression toward people and animals.

A 24-year-old woman presents to your office with a complaint of severe anxiety. Approximately three times per week she has episodes of sweating, chest pain, heart palpitations, shaking and fear of losing control or dying. The episodes seem to occur and resolve spontaneously. Which of the following is the most appropriate therapy? ACarbamazepine BCitalopram CImipramine DRisperidone

Correct Answer ( B ) Explanation: Panic disorder is a psychiatric illness characterized by recurrent episodes of panic attacks. Panic attacks are periods of intense fear with specific symptoms that develop quickly and peak less than 10 minutes from the onset of the attack. Symptoms include sweating, chest pain, heart palpitations, shaking, fear of losing control, fear of dying, perceived shortness of breath, nausea, dizziness, chills or hot flashes. Panic disorder often occurs co-morbidly with other psychiatric disorders, such as major depressive disorder, schizophrenia, obsessive-compulsive disorder, agoraphobia, and social phobia. Women experience panic disorder two to three times more frequently than men. Development of panic disorder generally occurs between ages 18-45 years, with the average age being 24 years. Diagnosis is determined using the Diagnostic and Statistical Manual of Mental Disorders, Fifth Edition (DSM-5) criteria. Initial treatment for panic disorder is with an antidepressant, cognitive behavioral therapy, or a combination of the two. When a decision is made to treat with medication, first-line treatment is with a selective serotonin reuptake inhibitor (SSRI), such as citalopram.

Which of the following is associated with an organic cause of psychosis? AAuditory hallucinations BDisorientation CNormal vital signs DSlow onset of symptoms

Correct Answer ( B ) Explanation: Psychosis can be caused by both organic (medical) and functional (psychiatric) etiologies. It is critical for the clinician to exclude organic causes of psychosis before transferring the patient to psychiatric services. The delay in diagnosis and therefore treatment is potentially harmful to the patient. Unfortunately, it can be difficult to differentiate the etiologies. Patients with organic causes of psychosis tend to have recent memory deficits, psychomotor retardation, visual hallucinations, emotional lability, disorientation, and occasional periods of lucidity. Additionally, those with organic psychosis are more likely to have a sudden onset of symptoms, abnormal vital signs or physical examination findings, and social immodesty. In patients over 40 years of age without a prior psychiatric history, an organic cause of psychosis should always be assumed.

A 43-year-old woman presents to the primary care office for concerns of sadness. She states that she has had some stress in her family recently. She has noticed, and her partner has mentioned, that her mood seems very labile and she usually seems sad. She struggles to gain interest in going to the gym, which she used to love to do daily. She feels tired most days for "no reason" as she gets 7-8 hours of sleep nightly. She is having a hard time concentrating at work and has noticed she is withdrawing from friendships. After discussion, the patient decides she would like to try medication. You are going to start her on the recommended first-line class of medications for the suspected diagnosis. What are the early side effects of the first-line medication that you should discuss with the patient? AHyperthermia, tachycardia, sedation BInsomnia, anxiety, headache CNausea, dizziness, diaphoresis DSedation, hyperlipidemia, hyperglycemia EWeight gain, dizziness, involuntary muscle twitches

Correct Answer ( B ) Explanation: Selective serotonin reuptake inhibitors (SSRIs) are recognized as the first-line pharmacotherapy for major depressive disorder. Besides major depressive disorder, they can also be helpful in treating panic disorder, obsessive-compulsive disorder, anxiety disorders, and eating disorders. Common SSRIs include citalopram, escitalopram, fluoxetine, fluvoxamine, paroxetine, and sertraline. They work by causing delayed serotonin reuptake at neurotransmitters in the brain. After starting treatment, it may take 2-4 weeks before the patient notices any benefit. Insomnia, anxiety, and headache are common early side effects that patients may experience. Sexual dysfunction is another common side effect. In overdoses with SSRIs, patients may develop serotonin syndrome. This is a potentially lethal condition due to overstimulation of serotonin receptors, which causes agitation, diaphoresis, tachycardia, hypertension, myoclonus, and hyperreflexia. Overall, SSRIs have a low side effect profile and are well tolerated by most patients over other antidepressant medication classes.

A 23-year-old man presents with leg pain for three months and requests hydromorphone for pain. The nurse approaches you because she believes the patient is "drug seeking" as he became extremely upset when she told him that he would have to wait for the doctor to evaluate him before pain medications could be given. Upon entering the room, the patient speaks pleasantly with you and compliments you on your kindness. He tells you that he only comes to this hospital because "it's the best in the world," and "none of the other doctors understand me." Upon informing the patient that you will not be prescribing hydromorphone, he becomes extremely upset and starts yelling. This patient is exhibiting traits that are consistent with which of the following personality disorders? AAntisocial BBorderline CHistrionic DNarcissistic

Correct Answer ( B ) Explanation: This patient exhibits a number of features typically seen in borderline personality disorder. Borderline personality disorder is characterized by unstable relationships, self image, and affect. This instability is often marked with impulsiveness. In the emergency department, these patients will often "split" providers: they will act with affection and respect to some providers and anger and disregard to others. Substance abuse and drug seeking behavior are often seen in borderline patients as well.

Which of the following medications is most likely to cause side effects such as tardive dyskinesia and have limited efficacy in treating the negative symptoms of schizophrenia? AAripiprazole BHaloperidol CQuetiapine DRisperidone

Correct Answer ( B ) Explanation: Typical antipsychotics such as haloperidol have a relatively high risk of adverse effects such as tardive dyskinesia and parkinsonism, especially in older patients. They are mainly effective against the positive symptoms. Antipsychotics are never to be given as an isolated measure; other forms of treatment, such as individual or family therapy, and psychosocial measures, should continue. Special education and support measures (compliance therapy) are useful where there are compliance problems. Schizophrenia is a chronic psychiatric disorder with no single feature that is pathognomic. Without treatment many patients may experience a downward drift in socioeconomic class. Second-generation atypical antipsychotics are generally associated with fewer extrapyramidal side effects, especially tardive dyskinesia, compared to first-generation antipsychotics. However there is debate about the efficacy of second and third-generation atypical antipsychotics efficacy for negative symptoms associated with schizophrenia. Aripiprazole (A), quetiapine (C), and risperidone (D), are all atypical antipsychotics.

You partner with a local school to create a checklist of symptoms that the teachers fill out to aid in the diagnosis of attention deficit hyperactivity disorder. Which combination of symptoms would be most consistent with a diagnosis of attention deficit hyperactivity disorder? AActively defies or refuses to follow teachers' rules, is easily annoyed by classmates and is frequently angry or resentful, argues with teachers BBullies or intimidates peers, leaves seat when remaining seated is expected, lies to get out of trouble, skips school without permission CHas difficulty organizing tasks and activities, blurts out answers before questions have been completed, has difficulty waiting for his or her turn, is forgetful in daily activities DIs easily distracted, frequently worries, is afraid to go to school or be called on by the teacher to answer questions

Correct Answer ( C ) Explanation: Attention deficit hyperactivity disorder is present when there is persistent pattern of inattention or hyperactivity-impulsivity that interferes with overall functioning. According to the DSM-V diagnostic criteria, symptoms must be present for at least six months and must be present prior to age 12 years. Additionally, symptoms are not better explained by another mental disorder. Symptoms consistent with a diagnosis of attention deficit hyperactivity disorder include difficulty organizing tasks and activities, blurting out answers before questions have been completed, having difficulty waiting for his or her turn, and being forgetful in daily activities.

Which of the following is true regarding attention-deficit/hyperactivity disorder? ACharacterized by speech delay, poor eye contact, and lack of stranger anxiety BMore prevalent in girls than boys CMost frequently diagnosed behavioral disorder in children DSymptoms most commonly begin in adolescence

Correct Answer ( C ) Explanation: Attention-deficit/hyperactivity disorder (ADHD) is the most frequently diagnosed behavioral disorder of childhood, with a prevalence of 4-12%. At least 10% of behavior problems seen in a general pediatric practice are caused by ADHD. ADHD is characterized by a triad of symptoms, including inattention, impulsivity, and hyperactivity. Symptoms must be present in two areas of social interaction (home and school), must have been present prior to age 12 years, must have persisted for longer than 6 months, and must be maladaptive or inappropriate for the child's developmental stage. Research suggests that ADHD has a central nervous system (CNS) basis, however, no specific etiology has been discovered. Various brain imaging studies of ADHD patients have demonstrated abnormalities of brain metabolism, supporting the validity of ADHD as a disorder. However, the strongest evidence of validity has been course prediction and treatment response to medication. There is no independent valid test to determine that a child has ADHD. The diagnosis can only be obtained reliably by using well-established diagnostic assessment methods. This involves using the standardized diagnostic criteria of the American Psychiatric Association's Diagnostic and Statistical Manual of Mental Disorders (DSM). Speech delay, poor eye contact, and lack of stranger anxiety (A) are characteristic of autism not ADHD. ADHD is more prevalent in boys than girls (B). Symptoms must have been present before the age of 12 (D).

A pregnant patient with a history of bipolar disorder presents to a prenatal appointment with irritability, rapid speech, and increased energy despite insomnia for 2 weeks. Prior to the pregnancy, the patient's mood symptoms were treated effectively with valproate, risperidone, and sertraline but she is no longer taking these. Administration of which one of the following is the most appropriate next step? AAripiprazole BHaloperidol CRisperidone DSertraline EValproate

Correct Answer ( C ) Explanation: Bipolar I disorder with peripartum onset describes manic symptoms beginning either during pregnancy or within the 4 weeks following delivery. Bipolar disorder is particularly important to recognize and treat, as untreated bipolar disorder during pregnancy is associated with poor birth outcomes and is a significant risk factor for the development of postpartum psychosis, which increases the risk of infanticide. Treatment guidelines for patients with bipolar disorder who are pregnant recommend using the lowest effective dose of agents that have worked in the past, as long as those medications are relatively safe in pregnancy. Switching or trying new medications adds a risk of inefficacy, thereby exposing the fetus to potential adverse effects of both the medication and the illness itself. Since this patient has developed manic symptoms, resuming the antimanic agent risperidone, which was previously effective, is the best next step in managing the symptoms. Second-generation antipsychotics, such as risperidone, olanzapine, and quetiapine, are effective antimanic agents and are not associated with an increased risk of malformations. Medications with lower potential for metabolic side effects, such as risperidone, aripiprazole, and lurasidone, are often recommended over olanzapine or quetiapine, which have more potential for weight gain and metabolic dysfunction. Although lithium has traditionally been associated with an increased risk of cardiac malformations, newer evidence suggests the risk may not be as high as previously thought. Therefore, if a patient has done well on lithium and is now pregnant, the benefit of continuing the medication might outweigh the risk. For patients with no prior trials of antimanic agents, starting either a first- or second-generation antipsychotic is recommended as first-line therapy. As the pregnancy progresses, dose increases are often necessary due to increased metabolism by the cytochrome P450 system. After delivery, doses should be reduced to preconception levels. For patients who are resistant to medication and experiencing severe dysfunction, suicidality, or psychosis, electroconvulsive therapy is also considered safe and effective in pregnancy.

A 67-year-old man is brought to the Emergency Department by his son and 70-year-old wife, who are concerned with his behavior. For the last six months, he has held a false belief that his wife is having sexual relationships with multiple men. His wife states she has not been sexually active for over ten years and denies the allegations. The son confirms he has been present with his mother when some of the alleged indiscretions have taken place and can confirm no other relationships are occurring. The patient has installed security cameras around the house in an attempt to "catch her in the act" and has not uncovered anything unusual. The patient is a recently retired detective, who explains that he is gathering evidence of his wife's infidelity. He denies alcohol and illicit drug use. He takes a statin to control his cholesterol, but is otherwise healthy. He has no prior mental health history and denies any manic or depressive symptoms. He denies hallucinations. His mood and affect are euthymic. His speech is organized and his behavior is polite. Which of the following is the most likely diagnosis? ABipolar disorder BBrief psychotic disorder CDelusional disorder DSchizophrenia

Correct Answer ( C ) Explanation: Delusional disorder is the presence of one or more delusions for a month or longer in a person who, except for the delusions and their behavioral ramifications, does not appear odd and is not functionally impaired. Prominent hallucinations and other psychotic or marked mood symptoms are absent. People with delusional disorder typically reject the characterization of their beliefs as false. They generally have no insight into their illness. Comorbidities are common, especially depression and anxiety. The onset of delusional disorder can be acute. Subtypes of delusional disorder include erotomanic, grandiose, jealous, persecutory, somatic, mixed, and unspecified types. First-line treatment of delusional disorder is usually with antipsychotic medication. Because many patients reject medication, psychotherapy is also appropriate.

Which of the following is true regarding anorexia nervosa? AAffects men more than women BAnti-depressants are an effective treatment CAssociated with a body image disturbance DMenorrhagia is an early symptom

Correct Answer ( C ) Explanation: Eating disorders are the third leading chronic illness, after obesity and asthma, in adolescent girls. An individual with anorexia nervosa refuses to maintain a minimally normal body weight, is fearful of gaining weight, and exhibits a distorted body self-image. The patient's body image is the predominant measure of self-worth, along with denial of the seriousness of the illness. The long-term mortality rate for anorexia nervosa is 6% to 20%, the highest rate for any psychiatric disorder. Anorexia is associated with amenorrhea, depression, fatigue, weakness, hair loss, bone pain, constipation, and abdominal pain. Signs include brittle hair and nails, dry, scaly skin, loss of subcutaneous fat, fine facial and body hair (lanugo), and breast and vaginal atrophy. A prime objective in assessment is to distinguish "normal dieters" from individuals with eating disorders. Another important aspect to the evaluation is to exclude certain medical conditions such as inflammatory bowel disease, hyperthyroidism, chronic infection, diabetes mellitus, and Addison disease. Indications for inpatient management include extremely low body weight (< 75% of expected) or rapid weight loss; severe electrolyte imbalances, cardiac disturbances, or other acute medical disorders; severe or intractable purging; psychosis or a high risk of suicide; and symptoms refractory to outpatient treatment. A multidisciplinary team that includes a primary physician, mental health professional, and registered dietician should manage patients with eating disorders. Refeeding should occur in a monitored setting due to the risk of dysrhythmia and refeeding syndrome.

A 60-year-old woman presents to your office accompanied by her adult daughter with a complaint of anxiety. She says that she's always been a "worrier," but since the birth of her grandchild last year her anxiety has gotten worse. Further discussion reveals that she sleeps only 2-3 hours per night, has daily headaches, and her daughter complains that she calls their house at least 15 times every day asking about her grandchild's safety. Which of the following is the most appropriate therapy? AImipramine BLorazepam CParoxetine DQuetiapine

Correct Answer ( C ) Explanation: Generalized anxiety disorder (GAD) is defined by having uncontrollable anxiety and worry that interfere with daily activities for at least 6 months. Symptoms occur on more days than not during this time period. First-line medications for GAD include SSRI's or SNRI's, including paroxetine. Psychotherapy is also recommended in the treatment of GAD. If one SSRI does not have the desired effect, then a trial of a different SSRI is used prior to initiating a second-line medication. Imipramine (A) is a tricyclic antidepressant that is considered second-line in the treatment of GAD. Lorazepam (B) is a benzodiazepine that can be used to augment SSRI therapy, especially in cases of agitation. Use of benzodiazepines is declining due to concerns about dependence and tolerance. Quetiapine (D) is an atypical antipsychotic that has been used in the treatment of GAD, however its use is off-label and it is not currently approved by the FDA.

A 36-year-old woman presents with 10/10 low back pain. She seems to be resting comfortably on the stretcher and is talking loudly on her cell phone. She describes in detail how she injured herself during sexual intercourse with a younger man. Her hair and makeup are impeccably done, and she is dressed in high-end, revealing clothing. She demands to be evaluated by a male doctor. She undresses in front of the clinician and is wearing lingerie. Which of the following personality disorders does this patient most likely have? AAntisocial BBorderline CHistrionic DNarcissistic

Correct Answer ( C ) Explanation: Individuals with histrionic personality disorder tend to be excessively emotional and demonstrate attention-seeking behavior. They want to be the center of attention. They often exaggerate (e.g., reporting 10/10 pain although resting comfortably and talking on the phone) and can be very flirtatious, overly dramatic, and sexually seductive (e.g., revealing lingerie in front of clinician, talking about sexual life). They rely on manipulative behavior to meet their needs. This disorder is more common in women and affects approximately 3% of the general population.

A 15-year-old girl presents to clinic for a well child check. During her Home and Environment, Education and Employment, Activities, Drugs, Sexuality, Suicide and Depression (HEADSS) exam, she reports using marijuana on a weekly basis for the past six months. She also drinks alcohol to excess once every few weeks. She denies smoking tobacco or using any other illicit substances. Which of the following is the most accurate information to give this patient? AChronic marijuana use improves focus and performance BIn human studies, marijuana has shown teratogenicity CMarijuana can cause tolerance and withdrawal DTetrahydrocannabinol concentrations are lower in today's marijuana compared to prior decades

Correct Answer ( C ) Explanation: Marijuana is the most commonly abused illicit drug, and is used by over 1/3 of US high school students. The active component, tetrahydrocannabinol (THC), leads to the effects of euphoria, elation, and hallucination. Side effects of the drug include impaired short-term memory, poor attention, loss of judgment, distorted time perception, and occasionally visual hallucinations and distorted body image. In the acute setting, serious adverse effects might include anxiety, panic, psychotic symptoms at high doses, and motor vehicle accidents. Marijuana is used in many medical conditions due to its antiemetic properties and appetite stimulation. Cannabis use disorder requires meeting at least 2 out of 11 possible criteria within a 12-month period (for example, tolerance and withdrawal) according to DSM-5 criteria. Withdrawal usually occurs within 24 to 48 hours of stopping the drug, and symptoms include malaise, irritability, insomnia, diaphoresis, night sweats, GI disturbance, and drug craving. The withdrawal symptoms usually peak by day 4 and are resolved by day 10-14.

A 20-year-old man presents to the primary care office with his parents, requesting an evaluation for attention-deficit/hyperactivity disorder. The patient has historically been a detail-oriented and high-achieving student. Since enrolling in college 2 years ago, the patient has been earning Cs in college. The patient does not understand why his parents are worried. He does not have any close friends or hobbies, in favor of devoting his time to creating detailed study guides, class schedules, and to-do lists. He spends 2-3 hours a day rewriting lists to ensure their accuracy. He recently failed a group project despite significant effort. He would not collaborate with his classmates because they did not work to his standards. The patient explains, "It's better to fail a project than to turn in something that is not up to par." What is the most likely diagnosis? ANarcissistic personality disorder BObsessive-compulsive disorder CObsessive-compulsive personality disorder DSchizoid personality disorder ESchizotypal personality disorder

Correct Answer ( C ) Explanation: Obsessive-compulsive personality disorder describes a pervasive preoccupation with orderliness, perfection, and control that begins by early adulthood and spans across multiple life domains. The symptoms are often experienced as ego-syntonic but, by definition, must cause distress or impairment. It is the most common personality disorder, affecting 5-8% of the US population. The etiology is not known, though biological, psychological, and sociocultural models of disease have been proposed. Risk factors may include male sex, insecure parental attachment, overcontrolling parents, and authoritarian cultural upbringing. The diagnosis is clinical and can be made using criteria from the fifth edition of the Diagnostic and Statistical Manual of Mental Disorders (DSM-5). The problematic pattern should have at least four of the following eight criteria: preoccupation with lists, details, rules, or organization in excess of utility; perfectionism that interferes with task completion; a devotion to work that excludes leisure and friendship; moral inflexibility that is not accounted for by culture; difficulty discarding worthless and unsentimental items; reluctance to delegate autonomous tasks to others; miserly saving to avoid catastrophes; and stubbornness. Psychotherapy is considered first-line therapy for all personality disorders. Medications may be used to treat comorbid illnesses or as adjunctive treatment to target specific symptom domains such as cognition and perception, affect, and behavior. Selective serotonin reuptake inhibitors such as fluoxetine and sertraline may be used as adjunctive treatment to target anxiety and cognitive rigidity.

A 38-year-old man presents to the office, reporting difficulty sleeping, with nightmares and anxiety for the past 2 months that have gotten more frequent. He returned from an overseas tour with the military 3 months ago. He also reports having flashbacks of his time in combat, especially when recalling his time there. The patient is alert and oriented but appears slightly withdrawn. A physical exam is unremarkable. Which of the following is the most appropriate pharmacotherapy for this patient in addition to immediate psychotherapy? ABuspirone BClonidine CParoxetine DTemazepam ETrazodone

Correct Answer ( C ) Explanation: Post-traumatic stress disorder (PTSD) is a syndrome characterized by intense thoughts and feelings that occur after experiencing a traumatic event (e.g., military combat, sexual violence, or severe injury) and last long after the event has ended. Symptoms include flashbacks, nightmares, avoidance of certain situations, negative thoughts and feelings, and hyperreactivity (e.g., being easily startled). Patients may also experience sleep problems such as nightmares, impulsivity, and difficulty concentrating. These symptoms may be triggered by reminders of the original traumatic event. Women are twice as likely as men to experience PTSD. The diagnosis is made when these symptoms persist for at least 1 month, according to the fifth edition of the Diagnostic and Statistical Manual of Mental Disorders. In a majority of cases of PTSD, there will be comorbid depression or panic disorder. Alcohol and substance use disorders are also associated comorbidities. Treatment includes psychotherapy initiated as soon as possible after the traumatic event to reduce the duration of symptoms. Cognitive processing therapy and prolonged exposure therapy are examples of approaches that help the individual face the traumatic situation and view it with less reactivity. Treatment of any comorbid substance use disorder is also important in helping the individual recover. In conjunction with psychotherapy, selective serotonin reuptake inhibitors (SSRIs) such as sertraline and paroxetine are the medications of choice for PTSD. They are helpful in improving depression, panic attacks, sleep problems, and startle responses. Other drugs such as beta-blockers (e.g., propranolol) help with peripheral symptoms of anxiety such as tremors and palpitations. Clonidine has been shown to help with hyperarousal symptoms, and prazosin has shown evidence of increasing quality of sleep and decreasing nightmares. Trazodone may also be used as a hypnotic agent. Benzodiazepines are contraindicated in the treatment of PTSD. The prognosis of this condition depends on early intervention after the trauma.

Which one of the following is highly suspicious for nonaccidental trauma in young pediatric patients? AClavicle fracture BDistal radius fracture CPosterior rib fracture DSpiral fracture of the tibia

Correct Answer ( C ) Explanation: Posterior rib fractures, metaphyseal fractures, multiple and bilateral fractures, fractures in different stages of healing, vertebral body fractures, fractures of the digits, scapular fractures, sternal fractures, and complex skull fractures are usually seen in child maltreatment or abuse. A skeletal survey should be obtained if suspicion for any of the above fractures is present as well as a consult to child protective services. Due to the greater pliability of children's ribs, greater force is required to produce a fracture and is usually caused by a direct blow to the ribs.

Katie is a 35-year-old woman who presents to the emergency department after being found wandering the streets yelling incoherently at imaginary people. After further investigation, you discover that she has been found arguing with an imaginary person for the past eight months and believes that she is being followed by the CIA. What is the most likely diagnosis? ADelusional disorder BSchizoaffective disorder CSchizophrenia DSchizophreniform disorder

Correct Answer ( C ) Explanation: Schizophrenia is a psychiatric disorder characterized by psychoses. Characteristic symptoms include hallucinations, delusions, disorganized speech or behavior. Negative symptoms must be present for at least six months in order to make the diagnosis. These symptoms begin to interfere with social, occupational, or self-care of a person. Treatment of schizophrenia is lifelong and requires both pharmacotherapy and psychotherapy.

A 25-year-old woman presents to the emergency department requesting help with detoxification from her substance use. She reports severe restlessness, anxiety, insomnia, and generalized myalgias in the past 24 hours since her last substance intake. Physical examination shows excessive lacrimation, diaphoresis, and piloerection. Withdrawal from which of the following substance is the most likely cause of this patient's condition? AAmphetamines BCocaine COpioids DPhencyclidine

Correct Answer ( C ) Explanation: Sufferers of opioid use disorder often begin by being prescribed opioid medication. Tolerance develops rapidly with long-term use of opioids and addiction can lead to drug-seeking behavior and experimentation with heroin. Opioid intoxication induces symptoms of euphoria followed by apathy, pinpoint pupils, constipation, drowsiness, slurred speech, and memory impairment. Overdose can lead to respiratory depression and coma and is treated with naloxone, an opioid antagonist. Opioid withdrawal is diagnosed when three or more of the following occur: dysphoric mood, nausea or vomiting, myalgias, lacrimation, rhinorrhea, pupillary dilation, piloerection, sweating, diarrhea, yawning, fever, and insomnia. Rapid detoxification from opioids is not life-threatening, but is extremely uncomfortable. Withdrawal is treated with supportive measures. Slower, tapered detoxification and withdrawal can be achieved using methadone (a mu agonist that is gradually titrated down) or buprenorphine (a partial mu agonist and antagonist that has a ceiling effect and long duration).

A 35-year old man believes he has colon cancer. He reports that his "peristalsis is louder than usual" and he has "excessive flatulence." He shows you a logbook that he has been keeping of his bowel habits for the last 6 months. He reports no weight loss, fatigue, night sweats, blood in his stool, or family history of colon cancer. He is having difficulty sleeping. What is the most likely diagnosis? ABody dysmorphic disorder BFunctional neurological symptom disorder CIllness anxiety disorder DSomatic symptom disorder

Correct Answer ( C ) Explanation: The patient has illness anxiety disorder. This was a new diagnosis that was introduced with the DSM-5, and it replaced reactive hypochondriasis. Patients that were previously diagnosed with reactive hypochondriasis are now classified as somatic symptom disorder or illness anxiety disorder. Most patients fall under the somatic symptom disorder diagnosis. If the patient reports predominantly physical symptoms, the patient has somatic symptom disorder. Patients with minimal physical symptoms and a greater focus on a possible disease have illness anxiety disorder. The patient will have excessive worry about having or acquiring a serious undiagnosed general medical disease. If there are physical symptoms present, it is typically due to an exaggeration of normal body functions. It is typically chronic in nature.

Which of the following represents the classic order of the stages of grief? AAnger, denial, bargaining, depression, acceptance BBargaining, anger, denial, depression, acceptance CDenial, anger, bargaining, depression, acceptance DDepression, denial, anger, bargaining, acceptance

Correct Answer ( C ) Explanation: The stages of mourning and grief represent a universal experience across cultures and backgrounds. The five stages of normal grief were first proposed by Elisabeth Kubler-Ross in 1969. People spend different amounts of time in each stage of grief. The first reaction to bad news is denial of the reality of the situation. Denial is a defense mechanism that buffers the immediate pain of shock. This is followed by anger that may be aimed at people or inanimate objects. It can often be directed at the doctor who delivered the news. Bargaining is the next stage and is a response to feelings of helplessness and vulnerability. Depression typically follows bargaining. There is often depression over the practical implications of the news as well as a deeper emotional depression. Finally, acceptance is reached. While this order is the classic teaching over time it has been found that people do not always progress through every stage nor necessarily in that order.

A 30-year-old woman presents to your office requesting medication to help her quit smoking. Which of the following is a contraindication to prescribing bupropion? ACardiovascular disease BChronic obstructive pulmonary disease CDepression DSeizure disorder

Correct Answer ( D ) Explanation: Bupropion is one of two pharmacologic agents used for smoking cessation, the other being varenicline. Bupropion is an antidepressant that is believed to enhance central nervous system noradrenergic and dopaminergic release. Patients start the medication one week prior to their quit date then continue the medication for eleven more weeks. Side effects of bupropion include headache, insomnia, agitation and dry mouth. Bupropion decreases the seizure threshold and is therefore contraindicated in patients with seizure disorder or any predisposition to seizures.

Which one of the following distinguishes conduct disorder from oppositional defiant disorder? AAngry and resentful BArgues with adults CDeliberately tries to annoy someone DPhysical aggression towards others

Correct Answer ( D ) Explanation: Conduct disorder can be distinguished from oppositional defiant disorder by the presence of physical aggression and other severe forms of antisocial behavior. Conduct disorder is characterized by a persistent pattern of serious rule-violating behavior, including behaviors that harm (or have the potential to harm) others. The patient with conduct disorder typically shows little concern for the rights or needs of others. The symptoms of conduct disorder are divided into four major categories: (1) physical aggression to people and animals including bullying, fighting, weapon carrying, cruelty to animals, and sexual aggression; (2) destruction of property including fire setting and breaking and entering; (3) deceitfulness and theft; and (4) serious rule violations including running away from home, staying out late at night without permission, and truancy. To meet the diagnosis, > three of these symptoms must be present at least 1 year (one or more in the past 6 months) and must impair the youth's function at home, at school, or with peers. The onset of conduct disorder may occur in early childhood but usually manifests in late childhood or adolescence. In the majority of patients, this disorder remits by adulthood. A substantial fraction of patients develops antisocial personality disorder as adults. Early onset of conduct disorder, along with a high frequency of diverse antisocial acts across multiple settings, predicts a worse prognosis and increased risk for antisocial personality disorder. Patients with conduct disorder are also at risk for the development of mood, anxiety, somatoform, and substance-use disorders in adulthood.

A 17-year-old girl presents to your clinic with a chief complaint of sadness, loss of appetite, and the inability to sleep. She is having a hard time focusing at work and she is tired all of the time. She tells you that for the past month she has been hearing voices telling her to harm herself. She is interested in treatment. What is the most appropriate first-line therapy for this condition? ALithium BPsychotherapy CSertraline DSertraline and olanzapine

Correct Answer ( D ) Explanation: Major depressive disorder is defined as an episode of major depression lasting at least two weeks with the following symptoms: (SIG E CAPS) sleep changes, interest (lack thereof), guilt, energy (lack thereof), cognition/concentration (lack thereof), appetite (increased or decreased), psychomotor (agitation or retardation), and suicide (thoughts of or attempts). Major depressive disorder with psychotic features is defined by the above mentioned symptoms plus psychotic features of delusions or hallucinations, either auditory or visual. The first-line treatment for major depressive disorder with psychotic features is an antidepressant plus an antipsychotic medication. Studies show that sertraline, an antidepressant, in combination with olanzapine, an antipsychotic, works more effectively than just sertraline or just olanzapine alone.

An 11-year-old boy is brought to the office by his mother for excessive daytime sleepiness. She states the patient sometimes falls asleep midconversation or while walking. He is sound asleep and rouses after 5-10 minutes. The patient reports occasional episodes of being awake but unable to move for several minutes as well. The patient also has an uncle who had similar symptoms when he was an adolescent. A physical exam is unremarkable. Which of the following medications is a first-line therapy for the treatment of this patient's condition and prominent symptom? AClonazepam BImipramine CMelatonin DModafinil EVenlafaxine

Correct Answer ( D ) Explanation: Narcolepsy is a condition of excessive sleepiness characterized by sudden involuntary transition into REM sleep at random and inappropriate times. It is thought to have some hereditary component. Symptoms include sudden sleep attacks that last several minutes and occur during any type of activity, cataplexy (sudden loss of muscle tone) that causes loss of mobility and is often associated with emotional reactions (e.g., anger, laughter), sleep paralysis, and visual or auditory hallucinations. This condition affects all sexes equally, begins early in life, and has variable severity, with symptoms tapering off in severity around 30 years of age. There is no cure for narcolepsy, and treatment focuses on symptom management and behavioral strategies. First-line pharmacologic therapy for daytime sleepiness is modafinil or armodafinil, which are wake-promoting medications. Additional first-line treatments include methylphenidate and dextroamphetamine, which are stimulants. There is some discrepancy in the literature on whether wake-promoting or stimulant medications should be used first in pediatric cases, more research is undergoing in this area. Serotonin and norepinephrine reuptake inhibitors (e.g., venlafaxine), selective serotonin reuptake inhibitors (e.g., fluoxetine), and tricyclic antidepressants (e.g., imipramine) are also used for cataplexy, sleep paralysis, and hallucinations but do not help treat excessive sleepiness. It is important that all patients be counseled on behavioral modifications and pharmacologic intervention. Having a regular sleep-wake schedule, getting adequate sleep at night, and taking scheduled daytime naps have been shown to help with drowsiness. Patients should avoid carbohydrate-rich meals, excessive sugar, and heavy meals since they can increase daytime drowsiness. Regular exercise can also improve alertness. Additionally, psychological counseling should be encouraged for the patient and their family to cope with this condition. Cognitive behavior therapy can also be used as a complement to pharmacotherapy and has been shown to promote treatment adherence.

An 18-year-old woman is concerned about heart palpitations, diaphoresis, nausea, and shaking of her whole body, along with a choking sensation. These symptoms have occurred three to four times per week for the last 2 weeks. She states that the symptoms are most common shortly before going to bed and begin suddenly peaking in 10 minutes and eventually spontaneously resolve. These symptoms are accompanied by an intense fear that she is going to die. Which of the following is the most likely diagnosis? AAngina BBipolar disorder CGeneralized anxiety disorder DPanic attack

Correct Answer ( D ) Explanation: Panic attacks are a collection of distressing physical, cognitive, and emotional symptoms that may occur in a variety of anxiety disorders, such as specific phobias, social phobias, post-traumatic stress disorder (PTSD), and acute stress disorder. Panic attacks are discrete periods of intense fear in the absence of real danger, accompanied by at least 4 of 13 cognitive and physical symptoms. The attacks have a sudden onset, build to a peak quickly, and are often accompanied by feelings of doom, imminent danger, and a need to escape. Symptoms of panic attacks can include somatic concerns (e.g., sweating, chills), cardiovascular symptoms (pounding heart, accelerated heart rate, chest pain), neurologic symptoms (trembling, unsteadiness, lightheadedness, paresthesias), GI symptoms (choking sensations, nausea), and pulmonary symptoms (shortness of breath). In addition, patients with panic attacks may worry they are dying, "going crazy," or have the sensation of being detached from reality. Patients with panic disorder experience recurrent, unexpected panic attacks, followed by at least 1 month of persistent worry that they will have another panic attack. Patients with panic disorder may begin to avoid places where a prior attack occurred or where help may not be available.

A 27-year-old woman presents with chest pain. The patient reports that she was about to give a presentation at her job and suddenly had a feeling of impending doom. In addition to chest pain, she had palpitations and sweating. Which of the following is this patient experiencing? AAcute stress disorder BAgoraphobia CHypomania DPanic attack

Correct Answer ( D ) Explanation: Panic attacks fall under the spectrum of anxiety disorders along with agoraphobia, obsessive-compulsive disorder, post-traumatic stress disorder (PTSD), and acute stress disorder. A panic attack is the sudden onset of symptoms including fear or terror often associated with a sense of impending doom. Anxiety disorders, in general, are caused by a combination of biopsychosocial factors and often interactions with situations of stress or trauma. Panic attacks can occur spontaneously without an expected trigger. The attacks are described as periods of significant fear followed by a constellation of symptoms that peak about 10 minutes after symptom onset. There are many manifestations of the attack including palpitations, sweating, shortness of breath, chest pain, nausea, dizziness, feelings of impending doom, numbness and tingling, and trembling.

A 34-year-old woman presents to the outpatient family practice office for evaluation. During her interview, she reports that she feels depressed and that this has been her baseline for the last eight or nine years. During this time, she has experienced occasional periods of happiness lasting one to two weeks at the most before returning to a sad or depressed state. She reports a generally low self esteem and indecisiveness which has made career advancement difficult. What is the most likely diagnosis? ABipolar I disorder BBipolar II disorder CMajor depressive disorder DPersistent depressive disorder

Correct Answer ( D ) Explanation: Persistent depressive disorder (previously known as dysthymic disorder), is characterized by at least two years of depressed mood on most days during adulthood. For this diagnosis to apply to children or adolescents, depressed mood on most days must persist for one year. During the two year period, the person cannot be symptom free for more than two months and must be free from a history of a major depressive episode. In addition, at least two of the following symptoms must be noted: poor concentration, indecisiveness, hopelessness, poor appetite, overeating, insomnia, hypersomnia, low energy, fatigue or lack of self esteem. This condition is more common in females and classically presents during young adulthood. A small percentage of patients will eventually meet the criteria for major depressive disorder. First-line treatment is a selective serotonin reuptake inhibitor. Second-line treatment is a serotonin-norepinephrine reuptake inhibitor. Bupropion or tricyclic antidepressants may also be trialed. Cognitive and insight-oriented therapy should also be offered.

Which of the following is an example of a positive symptom of schizophrenia? AAnxiety BApathy CImpaired memory DWord salad

Correct Answer ( D ) Explanation: Schizophrenia is a complex psychiatric syndrome in which there is impairment in several different domains. Positive symptoms of schizophrenia include hallucinations, delusions, and disorganization. Hallucinations may be auditory, visual, or both. Delusions are fixed false beliefs which may either be bizarre or non-bizarre. Disorganization may be evidenced by tangential speech, circumstantial speech, derailment, word salad (words thrown together without sensible meaning), or neologisms. Along with positive symptoms, patients with schizophrenia can also exhibit negative symptoms. While positive symptoms may be thought of as something being "added to" normal experience (e.g., hallucinations, delusions), negative symptoms are thought of as a deficit of normal experience (e.g., decreased emotional expressiveness, apathy, flat affect, and lack of energy, etc.). Yet another domain affected by the illness is cognition. Patients with schizophrenia suffer varying degrees of cognitive impairment, including decreased attention, decreased processing speed, decreased memory, decreased reasoning, difficulty interpreting social cues, and difficulty comprehending verbal communication. Mood and anxiety symptoms are also common in patients with schizophrenia. Finally, several physical manifestations of schizophrenia have been noted, including catatonia, metabolic disturbances (such as diabetes, dyslipidemia and hypertension), and neurologic disturbances (such as poor motor coordination and sensory integration). Schizophrenia often appears in the late teen to early adult years, may have an abrupt or an insidious onset and is considered a lifelong illness, although patients may experience periods of symptom remission. Treatment is with antipsychotic medication.

A wife is upset about how her husband has behaved after receiving a new diagnosis of terminal lung cancer. He has spent the last few days on the internet feverishly looking up his old girlfriends in an attempt to find phone numbers and call them. He has only slept a few hours in this time period, and has eaten only pretzels and soda. When the wife is finally able to get his attention, she asked him what he was doing, to which he responded "I feel great, so I thought I'd catch up with some old friends." He does not meet diagnostic criteria for a manic episode, and has no history of bipolar disorder. You suspect he may be in which stage of the Kubler-Ross grief reaction? AAnger BBargaining CCatastrophizing DDenial

Correct Answer ( D ) Explanation: The grief reaction has been described to occur in five stages by Elisabeth Kubler-Ross. This popular theory has not been tested though. It describes the emotions people experience when they are dealing with the death of a loved-one, catastrophic loss, tragedy or a new diagnosis of terminal illness. This staging system helps practitioners identify people who are grieving. Passage through the stages may not be linear, and some patients do not experience each stage. Denial is the first stage and acceptance is the fifth and final stage. Denial is a temporary attempt at personal defense, experienced consciously or subconsciously, in which a person downplays their symptoms ("I feel fine") or minimizes the situation ("This can't happen to me"). Usually people in the denial stage become ever more aware of their possessions and those who will survive their death. This can lead to a "reaching-out" to long lost relationships.

A 28-year-old woman presents to your office with a complaint of mood swings. She tells you that she constantly fights with her boyfriend and gets angry easily. She often uses cocaine and binge eats when under stress. Last month, she held a knife to her wrist during an argument with her boyfriend but did not follow through with the suicide attempt. Which of the following is the most appropriate next step in management? AAdmission to an inpatient psychiatric unit BBegin course of amitriptyline CBegin course of lorazepam DCognitive behavioral therapy

Correct Answer ( D ) Explanation: The hallmark features of borderline personality disorder (BPD) are instability of relationships, affect, and self-image, along with impulsive behaviors, including suicide attempts. BPD affects women more frequently than men, and patients often have co-morbid psychiatric disorders. The etiology of BPD is unknown but is believed to be a combination of psychosocial, biologic, and genetic factors. Diagnosis is made using the criteria in the Diagnostic and Statistical Manual of Mental Disorders (DSM-5), and BPD must be differentiated from other psychiatric disorders with similar characteristics. The manic behaviors of bipolar disorder may mimic the impulsivity seen in patients with BPD. First-line treatment for patients with BPD is psychotherapy, with different types of cognitive behavioral therapy being most effective.

You are seeing an 11-year-old girl for follow up of excessive handwashing and frequent worrying. You referred her for psychotherapy and she has been receiving cognitive behavioral therapy for 3 months with a child psychologist. However, her mother reports minimal improvement of her symptoms. The patient remains preoccupied with the thought of germs multiplying in her hands that may cause her to get sick. Her compulsion with handwashing is evident both at home and in school, causing impairment in school functioning as well as skin peeling and erythema of her hands. You decide that pharmacotherapy, in addition to cognitive behavioral therapy is the next best course of action. Which of the following is the most appropriate medication therapy to start? AAlprazolam BParoxetine CRisperidone DSertraline

Correct Answer ( D ) Explanation: The patient has symptoms of anxiety and obsessive-compulsive disorder which are impairing her overall functioning. In obsessive-compulsive disorder, the obsessions are dysfunctional and compulsions are out of the ordinary, and while associated with relief of distress reaction, these are not pleasurable to the individual. People with obsessive-compulsive disorder have insight about their condition. One way to differentiate fears and worries in anxiety disorders from obsessions is that they often pertain to real life situations, while obsessions typically involve some degree of irrationality. Although the first-line of treatment for anxiety and obsessive-compulsive disorder is cognitive behavioral therapy, use of medication with a selective serotonin reuptake inhibitor (SSRI) such as citalopram, fluoxetine or sertraline, in conjunction with cognitive behavioral therapy has been associated with a superior response to either treatment alone.

A 23-year-old man with a history of opiate use presents to the emergency department with tachycardia, hypertension, and mydriasis. Which of the following is true regarding management? AAntiemetics should be withheld as they may cause serious side effects BConsider ICU admission for the potential for symptoms to worsen CNaloxone should be given DSymptom control may be achieved with clonidine

Correct Answer ( D ) Explanation: This patient presents with opiate withdrawal and management should focus on the relief of symptoms. The onset of withdrawal symptoms depends on the opiate agent used. Heroin withdrawal typically occurs within 4-6 hours of discontinuation whereas methadone has a longer half-life and withdrawal may be delayed 24-48 hours. Withdrawal leads to sympathetic discharge and adrenergic hyperactivity. Symptoms include CNS excitation, tachypnea, and mydriasis. Tachycardia and hypertension are common. Additionally, patients will often complain of nausea, vomiting, and diarrhea. Physical examination may also reveal piloerection, yawning, rhinorrhea, and lacrimation. Opiate withdrawal is not life-threatening. Supportive and symptomatic care can decrease the patient's discomfort. Clonidine has been used to mitigate symptoms as have antiemetics.

A 53-year-old woman with a history of poorly controlled schizophrenia presented to the Emergency Department two weeks ago with altered mental status, auditory hallucinations, and agitation. During her inpatient psychiatry admission, she stabilized on clozapine. Which of the following laboratory tests should be performed at baseline and then weekly during treatment with clozapine? AFasting lipid panel BHemoglobin A1c CProlactin level DWhite blood cell count

Correct Answer ( D ) Explanation: White blood cell count is required due to the risk of life-threatening agranulocytosis. Clozapine is an antipsychotic medication used in patients with schizophrenia or schizoaffective disorder, who are resistant to other antipsychotic treatments. Patients being treated with clozapine must have a baseline white blood cell (WBC) count before initiating treatment and weekly throughout treatment. The US Food and Drug Administration requires patients in the United States to have a minimum absolute neutrophil count (ANC) level > 1500/microL to initiate treatment with clozapine.

A 28-year-old woman with bipolar disorder presents to the office reporting involuntary facial movements that have worsened over the past 3 months. She was prescribed risperidone and lithium about 6 months ago. On a physical exam, the patient has difficulty sticking out her tongue and has occasional involuntary movements of the tongue at rest. She also has increased blinking frequency. A blood lithium level is 0.9 mEq/L. Which of the following is the best next step in the management of her symptoms? AAdd benztropine as needed BAdd propranolol as needed CDiscontinue lithium and start valproic acid DIncrease dosage of risperidone and decrease lithium dosage ETaper and discontinue risperidone

Correct Answer ( E ) Explanation: Tardive dyskinesia is a condition characterized by abnormal movements of the face, mouth, tongue, trunk, and limbs due to long-term treatment with antipsychotic agents. Risk factors of developing these side effects include prolonged use of the offending drug, older age, tobacco use, and diabetes mellitus. Early symptoms include wormlike movements of the tongue at rest, difficulty in sticking out the tongue, facial tics, increased blinking, and abnormal jaw movements. As the syndrome progresses, there may be increased lip-smacking, chewing, opening and closing of the mouth, puffing of the cheeks, abnormal speech, respiratory distress, and involuntary movements throughout the body. Stress will often aggravate these symptoms. The dyskinesias do not occur during sleep and can sometimes be voluntarily suppressed for a short amount of time. Emphasis on treatment should be on the prevention of side effects by using the smallest amount of antipsychotic medications necessary to treat psychotic symptoms. When early manifestation of this condition occurs, gradually taper the patient off antipsychotic medications if possible and discontinue any anticholinergic medications that they may also be taking since these drugs worsen tardive dyskinesia. If psychotic episodes increase due to the lack of antipsychotic medication, it can be restarted at low doses and titrated upward until there is improvement in clinical symptoms. Risperidone and haloperidol are antipsychotics that are associated with a higher frequency of tardive dyskinesia, whereas clozapine and olanzapine are drugs of the same class that have a lower risk. Benzodiazepines or lithium, when used in conjunction with a low dose of antipsychotic medications, has been shown to help control psychotic symptoms. Benzodiazepines and other drugs, including buspirone, clonidine, calcium channel blockers, vitamin E, omega-3 fatty acids, and propranolol, are also minimally beneficial in treating dyskinesia side effects.

Which of the following medications is most likely to cause side effects such as tardive dyskinesia and have limited efficacy in treating the negative symptoms of schizophrenia? (A Aripiprazole B Haloperidol C Quetiapine D Risperidone)

Haloperidol (Typical antipsychotics such as haloperidol have a relatively high risk of adverse effects such as tardive dyskinesia and parkinsonism, especially in older patients. They are mainly effective against the positive symptoms. Antipsychotics are never to be given as an isolated measure; other forms of treatment, such as individual or family therapy, and psychosocial measures, should continue. Special education and support measures (compliance therapy) are useful where there are compliance problems. Schizophrenia is a chronic psychiatric disorder with no single feature that is pathognomic. Without treatment many patients may experience a downward drift in socioeconomic class. Second-generation atypical antipsychotics are generally associated with fewer extrapyramidal side effects, especially tardive dyskinesia, compared to first-generation antipsychotics. However there is debate about the efficacy of second and third-generation atypical antipsychotics efficacy for negative symptoms associated with schizophrenia. Aripiprazole (A), quetiapine (C), and risperidone (D), are all atypical antipsychotics.)

An 11-year-old boy is brought to the office by his mother for excessive daytime sleepiness. She states the patient sometimes falls asleep midconversation or while walking. He is sound asleep and rouses after 5-10 minutes. The patient reports occasional episodes of being awake but unable to move for several minutes as well. The patient also has an uncle who had similar symptoms when he was an adolescent. A physical exam is unremarkable. Which of the following medications is a first-line therapy for the treatment of this patient's condition and prominent symptom? (A Clonazepam B Imipramine C Melatonin D Modafinil E Venlafaxine)

Modafinil (Narcolepsy is a condition of excessive sleepiness characterized by sudden involuntary transition into REM sleep at random and inappropriate times. It is thought to have some hereditary component. Symptoms include sudden sleep attacks that last several minutes and occur during any type of activity, cataplexy (sudden loss of muscle tone) that causes loss of mobility and is often associated with emotional reactions (e.g., anger, laughter), sleep paralysis, and visual or auditory hallucinations. This condition affects all sexes equally, begins early in life, and has variable severity, with symptoms tapering off in severity around 30 years of age. There is no cure for narcolepsy, and treatment focuses on symptom management and behavioral strategies. First-line pharmacologic therapy for daytime sleepiness is modafinil or armodafinil, which are wake-promoting medications. Additional first-line treatments include methylphenidate and dextroamphetamine, which are stimulants. There is some discrepancy in the literature on whether wake-promoting or stimulant medications should be used first in pediatric cases, more research is undergoing in this area. Serotonin and norepinephrine reuptake inhibitors (e.g., venlafaxine), selective serotonin reuptake inhibitors (e.g., fluoxetine), and tricyclic antidepressants (e.g., imipramine) are also used for cataplexy, sleep paralysis, and hallucinations but do not help treat excessive sleepiness. It is important that all patients be counseled on behavioral modifications and pharmacologic intervention. Having a regular sleep-wake schedule, getting adequate sleep at night, and taking scheduled daytime naps have been shown to help with drowsiness. Patients should avoid carbohydrate-rich meals, excessive sugar, and heavy meals since they can increase daytime drowsiness. Regular exercise can also improve alertness. Additionally, psychological counseling should be encouraged for the patient and their family to cope with this condition. Cognitive behavior therapy can also be used as a complement to pharmacotherapy and has been shown to promote treatment adherence. Imipramine (B) is a tricyclic antidepressant used for the treatment of cataplexy, sleep paralysis, and hallucinations but does not help with excessive sleepiness. Melatonin (C) is an over the counter dietary supplement used for the short-term treatment of insomnia. Venlafaxine (E) is a serotonin and norepinephrine reuptake inhibitor used for the treatment of cataplexy, sleep paralysis, and hallucinations but does not help with excessive sleepiness.)

Question: What is the duration of action for naloxone?

Naloxone will reverse opiate effects for 1-2 hours.

A 25-year-old woman presents to the emergency department requesting help with detoxification from her substance use. She reports severe restlessness, anxiety, insomnia, and generalized myalgias in the past 24 hours since her last substance intake. Physical examination shows excessive lacrimation, diaphoresis, and piloerection. Withdrawal from which of the following substance is the most likely cause of this patient's condition? (A Amphetamines B Cocaine C Opioids D Phencyclidine)

Opioids (Sufferers of opioid use disorder often begin by being prescribed opioid medication. Tolerance develops rapidly with long-term use of opioids and addiction can lead to drug-seeking behavior and experimentation with heroin. Opioid intoxication induces symptoms of euphoria followed by apathy, pinpoint pupils, constipation, drowsiness, slurred speech, and memory impairment. Overdose can lead to respiratory depression and coma and is treated with naloxone, an opioid antagonist. Opioid withdrawal is diagnosed when three or more of the following occur: dysphoric mood, nausea or vomiting, myalgias, lacrimation, rhinorrhea, pupillary dilation, piloerection, sweating, diarrhea, yawning, fever, and insomnia. Rapid detoxification from opioids is not life-threatening, but is extremely uncomfortable. Withdrawal is treated with supportive measures. Slower, tapered detoxification and withdrawal can be achieved using methadone (a mu agonist that is gradually titrated down) or buprenorphine (a partial mu agonist and antagonist that has a ceiling effect and long duration))

Which one of the following is highly suspicious for nonaccidental trauma in young pediatric patients? (Clavicle fracture Distal radius fracture Posterior rib fracture Spiral fracture of the tibia)

Posterior rib fracture (Posterior rib fractures, metaphyseal fractures, multiple and bilateral fractures, fractures in different stages of healing, vertebral body fractures, fractures of the digits, scapular fractures, sternal fractures, and complex skull fractures are usually seen in child maltreatment or abuse. A skeletal survey should be obtained if suspicion for any of the above fractures is present as well as a consult to child protective services. Due to the greater pliability of children's ribs, greater force is required to produce a fracture and is usually caused by a direct blow to the ribs.)

A pregnant patient with a history of bipolar disorder presents to a prenatal appointment with irritability, rapid speech, and increased energy despite insomnia for 2 weeks. Prior to the pregnancy, the patient's mood symptoms were treated effectively with valproate, risperidone, and sertraline but she is no longer taking these. Administration of which one of the following is the most appropriate next step? (A Aripiprazole B Haloperidol C Risperidone D Sertraline E Valproate)

Risperidone (Bipolar I disorder with peripartum onset describes manic symptoms beginning either during pregnancy or within the 4 weeks following delivery. Bipolar disorder is particularly important to recognize and treat, as untreated bipolar disorder during pregnancy is associated with poor birth outcomes and is a significant risk factor for the development of postpartum psychosis, which increases the risk of infanticide. Treatment guidelines for patients with bipolar disorder who are pregnant recommend using the lowest effective dose of agents that have worked in the past, as long as those medications are relatively safe in pregnancy. Switching or trying new medications adds a risk of inefficacy, thereby exposing the fetus to potential adverse effects of both the medication and the illness itself. Since this patient has developed manic symptoms, resuming the antimanic agent risperidone, which was previously effective, is the best next step in managing the symptoms. Second-generation antipsychotics, such as risperidone, olanzapine, and quetiapine, are effective antimanic agents and are not associated with an increased risk of malformations. Medications with lower potential for metabolic side effects, such as risperidone, aripiprazole, and lurasidone, are often recommended over olanzapine or quetiapine, which have more potential for weight gain and metabolic dysfunction. Although lithium has traditionally been associated with an increased risk of cardiac malformations, newer evidence suggests the risk may not be as high as previously thought. Therefore, if a patient has done well on lithium and is now pregnant, the benefit of continuing the medication might outweigh the risk. For patients with no prior trials of antimanic agents, starting either a first- or second-generation antipsychotic is recommended as first-line therapy. As the pregnancy progresses, dose increases are often necessary due to increased metabolism by the cytochrome P450 system. After delivery, doses should be reduced to preconception levels. For patients who are resistant to medication and experiencing severe dysfunction, suicidality, or psychosis, electroconvulsive therapy is also considered safe and effective in pregnancy.)

A 17-year-old girl presents to your clinic with a chief complaint of sadness, loss of appetite, and the inability to sleep. She is having a hard time focusing at work and she is tired all of the time. She tells you that for the past month she has been hearing voices telling her to harm herself. She is interested in treatment. What is the most appropriate first-line therapy for this condition? (A Lithium B Psychotherapy C Sertraline D Sertraline and olanzapine)

Sertraline and olanzapine (Major depressive disorder is defined as an episode of major depression lasting at least two weeks with the following symptoms: (SIG E CAPS) sleep changes, interest (lack thereof), guilt, energy (lack thereof), cognition/concentration (lack thereof), appetite (increased or decreased), psychomotor (agitation or retardation), and suicide (thoughts of or attempts). Major depressive disorder with psychotic features is defined by the above mentioned symptoms plus psychotic features of delusions or hallucinations, either auditory or visual. The first-line treatment for major depressive disorder with psychotic features is an antidepressant plus an antipsychotic medication. Studies show that sertraline, an antidepressant, in combination with olanzapine, an antipsychotic, works more effectively than just sertraline or just olanzapine alone.)

Question: What type of fracture of the elbow is a child most likely to sustain by falling from a jungle gym?

Supracondylar fractures are the most common pediatric elbow fracture and occur after a fall from a jungle gym or a tree.

A 23-year-old man with a history of opiate use presents to the emergency department with tachycardia, hypertension, and mydriasis. Which of the following is true regarding management? ( Antiemetics should be withheld as they may cause serious side effects Consider ICU admission for the potential for symptoms to worsen Naloxone should be given Symptom control may be achieved with clonidine)

Symptom control may be achieved with clonidine (This patient presents with opiate withdrawal and management should focus on the relief of symptoms. The onset of withdrawal symptoms depends on the opiate agent used. Heroin withdrawal typically occurs within 4-6 hours of discontinuation whereas methadone has a longer half-life and withdrawal may be delayed 24-48 hours. Withdrawal leads to sympathetic discharge and adrenergic hyperactivity. Symptoms include CNS excitation, tachypnea, and mydriasis. Tachycardia and hypertension are common. Additionally, patients will often complain of nausea, vomiting, and diarrhea. Physical examination may also reveal piloerection, yawning, rhinorrhea, and lacrimation. Opiate withdrawal is not life-threatening. Supportive and symptomatic care can decrease the patient's discomfort. Clonidine has been used to mitigate symptoms as have antiemetics.)

A 28-year-old man presents to the emergency department by ambulance. His family called for help after finding him unresponsive at home with a syringe on the floor beside him. His blood pressure is 120/78 mm Hg, pulse 95/min, and respirations are 6/min and shallow. On physical exam he is non-responsive to questions, his skin is cool with cyanosis, and his pupils are minimally reactive to light and constricted. Which of the following is the most appropriate next step in management? AAdminister naloxone BAdminister sodium bicarbonate CPlace a nasogastric tube and administer activated charcoal DSupport airway and breathing

Correct Answer ( D ) Explanation: Opioid abuse and overdose is a problem in the United States and worldwide. Deaths due to overdose are common and increasing in number. Clinical features of opioid intoxication include altered mental status, hypoventilation, decreased bowel sounds, low to normal blood pressure and heart rate, and miotic pupils. Medical providers should attempt to obtain as much historical information as possible, however an accurate history is not essential in initial management of these patients. Hypoventilation is the most common vital sign abnormality. The first step in management of an overdose is supporting the patient's airway by providing assisted ventilation with supplemental oxygen through the use of a bag-valve-mask.

A 16-year-old girl presents to the ED via ambulance for generalized pain. She is a refugee from a conflict area who is known to have frequent nighttime visits to the ED over the past year for the same chief concern. She has undergone multiple medical workups that have all been negative. In the ED, she appears angry, irritable, and she demonstrates hypervigilance. After a brief conversation with the patient, her pain resolves, and she feels much better. Which of the following is the most likely diagnosis? AAdjustment disorder BBorderline personality disorder CMalingering DPosttraumatic stress disorder

Correct Answer ( D ) Explanation: The patient in the above clinical scenario is a refugee from a conflict region and exhibits symptoms consistent with post-traumatic stress disorder (PTSD), including anger, irritability, and hypervigilance. PTSD is a long-lasting anxiety response following a traumatic or catastrophic event. Although most people encounter trauma over a lifetime, about 20-30% develop PTSD, and over half of these people will recover without treatment. Prediction models have consistently found that childhood trauma, chronic adversity, and familial stressors increase the risk of developing PTSD. Other risk factors include military experiences, war-zone exposure, domestic violence, and foster care. PTSD often leads to patients having difficulty falling or staying asleep, problems with concentration, hypervigilance, irritability, angry outbursts, and increased startle response. An important management principle when caring for a patient with PTSD is to ensure their safety and validate their symptoms. Detailed questioning should be avoided as it may trigger severe symptoms.

A 43-year-old woman presents to the primary care office for concerns of sadness. She states that she has had some stress in her family recently. She has noticed, and her partner has mentioned, that her mood seems very labile and she usually seems sad. She struggles to gain interest in going to the gym, which she used to love to do daily. She feels tired most days for "no reason" as she gets 7-8 hours of sleep nightly. She is having a hard time concentrating at work and has noticed she is withdrawing from friendships. After discussion, the patient decides she would like to try medication. You are going to start her on the recommended first-line class of medications for the suspected diagnosis. What are the early side effects of the first-line medication that you should discuss with the patient? (A Hyperthermia, tachycardia, sedation B Insomnia, anxiety, headache C Nausea, dizziness, diaphoresis D Sedation, hyperlipidemia, hyperglycemia E Weight gain, dizziness, involuntary muscle twitches)

Insomnia, anxiety, headache (Selective serotonin reuptake inhibitors (SSRIs) are recognized as the first-line pharmacotherapy for major depressive disorder. Besides major depressive disorder, they can also be helpful in treating panic disorder, obsessive-compulsive disorder, anxiety disorders, and eating disorders. Common SSRIs include citalopram, escitalopram, fluoxetine, fluvoxamine, paroxetine, and sertraline. They work by causing delayed serotonin reuptake at neurotransmitters in the brain. After starting treatment, it may take 2-4 weeks before the patient notices any benefit. Insomnia, anxiety, and headache are common early side effects that patients may experience. Sexual dysfunction is another common side effect. In overdoses with SSRIs, patients may develop serotonin syndrome. This is a potentially lethal condition due to overstimulation of serotonin receptors, which causes agitation, diaphoresis, tachycardia, hypertension, myoclonus, and hyperreflexia. Overall, SSRIs have a low side effect profile and are well tolerated by most patients over other antidepressant medication classes. Hyperthermia, tachycardia, and sedation (A) are symptoms of tricyclic antidepressant overdose. Tricyclic antidepressants inhibit the reuptake of norepinephrine and serotonin. In overdose, they cause anticholinergic effects with hemodynamic decompensation leading to tachycardia and hyperthermia. Sedation is secondary to antagonism of histamine 1 receptors. Patients typically are stable at first, but then decompensate rapidly after overdose. Treatment includes supportive care as well as sodium bicarbonate for any cardiac instability. Benzodiazepines are also helpful in treatment. Nausea, dizziness, and diaphoresis (C) are common side effects of serotonin and norepinephrine reuptake inhibitors (SNRIs). SNRIs act on serotonergic and noradrenergic neurons. They are used in the treatment of major depression, anxiety disorders, and panic disorder. In addition to the side effects listed above, they may cause hypertension, headaches, and sexual dysfunction. Sedation, hyperlipidemia, and hyperglycemia (D) are side effects of atypical or second-generation antipsychotics. The mechanism of cause for hyperlipidemia and hyperglycemia is not well known, but patients with underlying risk factors have a greater likelihood to develop hyperlipidemia and hyperglycemia. Sedation is secondary to the second-generation antipsychotics being H1 receptor antagonists leading to drowsiness. Weight gain, dizziness, and involuntary muscle twitches (E) are side effects that can be seen with the use of monoamine oxidase inhibitors (MAOIs). Monoamine oxidase inhibitors are used to treat refractory depression. They can have significant side effects, drug to drug interactions, and are much more lethal in overdose than other antidepressants. Their mechanism of action is not understood well, but may be due to increased serotonergic, dopaminergic, and noradrenergic transmission.)

Question: What is alcohol withdrawal delirium (delirium tremens)?

Delirium and clouded consciousness that occurs as part of alcohol withdrawal.

Question: What are the most common methods of completed suicides and suicide attempts?

Most completed suicides involve firearms, and most suicide attempts involve ingestions.

Question: What is the most common side effect of olanzapine?

Weight gain.


Kaugnay na mga set ng pag-aaral

English 12A - Unit Five: For the Sake of Argument

View Set

Unit 2: Security Solutions for Wi-Fi Networks

View Set

Industrialization and the British Empire

View Set

IMTA Music Theory Terms - Levels A-F

View Set

Greek Religion: key modern scholarship

View Set

Chapter 13: Financial Statement Analysis

View Set

Module 5 - Networking and Content Delivery [Knowledge Check], Module 4 - AWS Cloud Security [Knowledge Check], Module 3 - AWS Global Infrastructure Overview [Knowledge Check], Module 2 - Cloud Economics and Billing [Knowledge Check], AWS Module 1-6

View Set

Chapter 39: The Child with a Genitourinary Disorder Due

View Set

Federal Tax Considerations: Life Insurance

View Set

Parkinsons disease & Multiple sclerosis

View Set